Difference between revisions of "2024 AMC 10A Problems/Problem 18"
(→Problem) |
Mathkiddus (talk | contribs) (→Solution) |
||
Line 5: | Line 5: | ||
==Solution== | ==Solution== | ||
+ | <math>2b^3+2b+4\equiv 0\pmod{16}\implies b^3+b+2\equiv 0\pmod 8</math>, if <math>b</math> even then <math>b+2\equiv 0\pmod 8\implies b\equiv 6\pmod 8</math>. If <math>b</math> odd then <math>b^2\equiv 1\pmod 8\implies b^3+b+2\equiv 2b+2\pmod 8</math> so <math>2b+2\equiv 0\pmod 8\implies b+1\equiv 0\pmod 4\implies b\equiv 3,7\pmod 8</math>. Now <math>8\mid 2024</math> so <math>\tfrac38\cdot 2024=759</math> but <math>3</math> is too small so <math>758\implies\boxed{20}</math>. | ||
+ | ~OronSH ~mathkiddus |
Revision as of 16:03, 8 November 2024
Problem
There are exactly positive integers with such that the base- integer is divisible by (where is in base ten). What is the sum of the digits of ?
Solution
, if even then . If odd then so . Now so but is too small so . ~OronSH ~mathkiddus